Last visit was: 26 Apr 2024, 23:53 It is currently 26 Apr 2024, 23:53

Close
GMAT Club Daily Prep
Thank you for using the timer - this advanced tool can estimate your performance and suggest more practice questions. We have subscribed you to Daily Prep Questions via email.

Customized
for You

we will pick new questions that match your level based on your Timer History

Track
Your Progress

every week, we’ll send you an estimated GMAT score based on your performance

Practice
Pays

we will pick new questions that match your level based on your Timer History
Not interested in getting valuable practice questions and articles delivered to your email? No problem, unsubscribe here.
Close
Request Expert Reply
Confirm Cancel
SORT BY:
Kudos
Tags:
Show Tags
Hide Tags
Math Expert
Joined: 02 Sep 2009
Posts: 92948
Own Kudos [?]: 619294 [5]
Given Kudos: 81609
Send PM
GMATWhiz Representative
Joined: 07 May 2019
Posts: 3409
Own Kudos [?]: 1800 [3]
Given Kudos: 68
Location: India
GMAT 1: 740 Q50 V41
GMAT 2: 760 Q51 V40
Send PM
GMAT Club Legend
GMAT Club Legend
Joined: 08 Jul 2010
Status:GMAT/GRE Tutor l Admission Consultant l On-Demand Course creator
Posts: 5962
Own Kudos [?]: 13392 [1]
Given Kudos: 124
Location: India
GMAT: QUANT+DI EXPERT
Schools: IIM (A) ISB '24
GMAT 1: 750 Q51 V41
WE:Education (Education)
Send PM
Senior Moderator - Masters Forum
Joined: 19 Jan 2020
Posts: 3137
Own Kudos [?]: 2769 [1]
Given Kudos: 1510
Location: India
GPA: 4
WE:Analyst (Internet and New Media)
Send PM
Re: A telecommunications companys stock fell 10% last month, while a riva [#permalink]
1
Kudos
Let the telecommunications company’s stock value at the beginning of last month be x which fell 10% last month = 0.9x

And rival company’s stock at the beginning of last month is Y, which increased by 10% = 1.1Y

The rival company’s stock value today is what percentage of the telecommunication company’s stock value at the beginning of last month = 1.1y is what percentage of X

Statement 1: The rival company’s stock was valued at $42 per share at the beginning of last month.
Y = 42
Increase = 42*1.1 = 46.2
We need to know 46.2 is what percentage of X
=> 46.2*100/X
No value of X (insufficient)

Statement 2: The dollar amount of the rival company’s stock increase was 90% as much as the dollar amount of the decrease in the telecommunication company’s stock.
Increase in rival stock = 0.1Y
Decrease in telecommunication company stock = 0.1x
0.1y = 0.9*0.1x
0.1y = 0.09x
X/y = 1.111/1 or y/x = 0.9
We want the value of 1.1y/x*100
Putting values.
1.1*y/x*100
1.1*0.9 = 0.99
(Sufficient)

Answer is B

Posted from my mobile device
Manager
Manager
Joined: 24 Sep 2013
Posts: 164
Own Kudos [?]: 109 [1]
Given Kudos: 40
Location: Saudi Arabia
GPA: 3.8
WE:Project Management (Energy and Utilities)
Send PM
Re: A telecommunications companys stock fell 10% last month, while a riva [#permalink]
1
Kudos
TC stock fell last month = 10% ; Today, the stock value of the telecommunication company =0.9x=0.9x
rival's stock increased last month = 10% ; And, the stock value of it’s rival company, today =1.1y=1.1y


1.1y = m% * 0.9x
m=?

St1: rivals stock value, y = $42
The value of "x" we don't know ; Not Suff.

St2: 0.1*y = 0.9* 0.1 * x
Thus we can find the value of y / x = 0.9
Sufficient


Thus, the correct answer is Option B.
Stern School Moderator
Joined: 26 May 2020
Status:Spirited
Posts: 634
Own Kudos [?]: 539 [1]
Given Kudos: 219
Concentration: General Management, Technology
WE:Analyst (Computer Software)
Send PM
Re: A telecommunications companys stock fell 10% last month, while a riva [#permalink]
1
Kudos
Bunuel wrote:
A telecommunications company’s stock fell 10% last month, while a rival company’s stock increased by 10%. The rival company’s stock value today is what percentage of the telecommunication company’s stock value at the beginning of last month?

(1) The rival company’s stock was valued at $42 per share at the beginning of last month.

(2) The dollar amount of the rival company’s stock increase was 90% as much as the dollar amount of the decrease in the telecommunication company’s stock.


Ans : B , imo

------------------ ----------Last Month----------------------This month-------

--Telecom Stock -----------S--------------------------------0.9 S ---

--Rival Stock---------------R--------------------------------1.1 R----

Ques is (1.1 R / S ) * 100 = ?

So if we get the value of R?S.. we can get the final value .



Statement 1: The rival company’s stock was valued at $42 per share at the beginning of last month.

No info about S . so NOT SUFFICIENT

Statement 2: The dollar amount of the rival company’s stock increase was 90% as much as the dollar amount of the decrease in the telecommunication company’s stock.

0.1R = 0.9 (0.1)S
So we can find R/S and the final value . Hence SUFFICIENT

So ans is B .
CEO
CEO
Joined: 07 Mar 2019
Posts: 2555
Own Kudos [?]: 1813 [1]
Given Kudos: 763
Location: India
WE:Sales (Energy and Utilities)
Send PM
Re: A telecommunications companys stock fell 10% last month, while a riva [#permalink]
1
Kudos
A telecommunications company’s stock fell 10% last month, while a rival company’s stock increased by 10%. The rival company’s stock value today is what percentage of the telecommunication company’s stock value at the beginning of last month?

Price of Company stock at the start of month, \(P_c = x\)
Price of Company stock at the end of month, \(P_c' = 0.9x\)
Price of rival Company stock at the start of month, \(P_{rc} = y\)
Price of rival Company stock at the end of month, \(P_{rc}' = 1.1y\)

\(\frac{P_{rc}' - P_c}{P_c} * 100 = \frac{1.1 y - x}{x} * 100\) = ?
\(\frac{P_{rc}'}{P_c} = \frac{1.1 y}{x} = ?\)
\(\frac{y}{x} = ?\)

(1) The rival company’s stock was valued at $42 per share at the beginning of last month.
y = 42
x = ?

INSUFFICIENT.

(2) The dollar amount of the rival company’s stock increase was 90% as much as the dollar amount of the decrease in the telecommunication company’s stock.
1.1y - y = 0.9 * (x - 0.9x)
0.1y = 0.9 * 0.1x
\(\frac{y}{x} = 0.9\)
\(\frac{P_{rc}'}{P_c} = 1.1 * 0.9 = 0.99\)

SUFFICIENT.

Answer B.
Senior Manager
Senior Manager
Joined: 05 Aug 2019
Posts: 317
Own Kudos [?]: 279 [0]
Given Kudos: 130
Location: India
Concentration: Leadership, Technology
GMAT 1: 600 Q50 V22
GMAT 2: 670 Q50 V28 (Online)
GPA: 4
Send PM
Re: A telecommunications companys stock fell 10% last month, while a riva [#permalink]
Kindly see the attachment.
Need to find the ratio of prices of TC and rival company
B
Attachments

1.PNG
1.PNG [ 22.22 KiB | Viewed 2076 times ]

GMAT Club Bot
Re: A telecommunications companys stock fell 10% last month, while a riva [#permalink]
Moderator:
Math Expert
92948 posts

Powered by phpBB © phpBB Group | Emoji artwork provided by EmojiOne